Kit and Kat are building a kite for the big kite festival. Kit has already
cut his sticks for the diagonals. He wants to position P so that he will
have maximum kite area. He asks Kat for advice. What should Kat
tell him?

Kit And Kat Are Building A Kite For The Big Kite Festival. Kit Has Alreadycut His Sticks For The Diagonals.

Answers

Answer 1

Kat should tell Kim that It's too late to change the area of this kite because the length of the diagonals of a kite determines its area.

How to calculate the area of a kite?

In Mathematics, the area of a kite is equal to one-half the product of the length of its diagonals. Mathematically, the area of a kite can be calculated by using this formula:

A = ½ × d₁ × d₂

Where:

A is the area of a kite.d₁ and d₂ are the length of the diagonals of a kite.

Since Kit has already cut his sticks for the diagonals, Kat should tell him that it's too late to change the area of this kite because the length of the diagonals of a kite determines its area.

Read more on area of a kite here: https://brainly.com/question/9593985

#SPJ1


Related Questions

need help asap! Fill in the blank by entering just a number…

Answers

Answer:

x = 6

Step-by-step explanation:

CT is a median, which means it divided AB into two equal parts so we can write the following equation:

7x + 9 = 5x + 21 transfer like terms to the same side of the equation

7x - 5x = 21 - 9 do the extraction

2x = 12 divide both sides by 2

x = 6

Write [tex]\frac{1}{22}[/tex] as a decimal, Also please include the working out plus anyone that includes other examples will be given brainlest!

Thank you!

Answers

Do long division (see the attachment).

[tex]\dfrac{1}{22}=0.04545\ldots=0.0\overline{45}[/tex]

Find the point Q along the directed line segement from point R (-3, 3) to point S(6, -3) that divides the segment in the ratio 2:1.

A. (3, 0)
B. (6, -4)
C. (0, 1)
D. (3, -1)

Answers

Answer: (3,-1)

Step-by-step explanation:

distance between R & S x values: 9

distance between R & S y values: -6

multiply each by 2/3 (the proportion)

9/1 x 2/3 = 6; -6/1 x 2/3= -4

add the answers to R x & y values:

-3 + 6= 3; 3 - 4= -1

Answer:

D

Step-by-step explanation:

using the section formula

(x₁, y₁ ) and (x₂, y₂ ) divided in the ratio m : n , then

point = ([tex]\frac{mx_{2}+nx_{1} }{m+n}[/tex] , [tex]\frac{my_{2}+ny_{1} }{m+n}[/tex] )

thus for R (- 3, 3 ) and S (6, - 3 ) in the ratio 2 : 1

Q = ( [tex]\frac{2(6)+1(-3)}{2+1}[/tex] , [tex]\frac{2(-3)+1(3)}{2+1}[/tex] )

   = ( [tex]\frac{12-3}{3}[/tex] , [tex]\frac{-6+3}{3}[/tex] )

  = ( [tex]\frac{9}{3}[/tex] , [tex]\frac{-3}{3}[/tex] )

  = (3, - 1 )

Find the nth term of the sequence 3, -1, -5, -9.

Answers

Answer:

-4n+7

Step-by-step explanation:

because you take away 4 each time and the nber that would come before 3 is 7.

5. The mapping f: xax² + bx + c defined on the set of real numbers is such that f(0) = -4,f(1)-1 and/(-1)= -5. Find a, b and c. 5 . The mapping f : xax² + bx + c defined on the set of real numbers is such that f ( 0 ) = -4 , f ( 1 ) -1 and / ( - 1 ) = -5 . Find a , b and c .​

Answers

It looks like you're saying

[tex]f(x) = ax^2 + bx + c[/tex]

and you're asked to find [tex]a,b,c[/tex] given [tex]f(0)=-4[/tex], [tex]f(1)=-1[/tex], and [tex]f(-1)=-5[/tex].

Evaluate [tex]f[/tex] at the three given points:

[tex]x=0 \implies f(0) = \boxed{c = -4}[/tex]

[tex]x=1 \implies f(1) = -1 = a + b + c \implies a+b = 3[/tex]

[tex]x=-1 \implies f(-1) = -5 = a - b + c \implies a - b = -1[/tex]

[tex](a+b) + (a-b) = 3 + (-1) \implies 2a = 2 \implies \boxed{a=2}[/tex]

[tex]a-b = -1 \implies \boxed{b=3}[/tex]

and the mapping is [tex]f(x) = 2x^2 + 3x - 4[/tex].


The question is:
Jason has 20 gallons of a 12% alcohol solution. How many gallons, to the nearest tenth, must be replaced by a 36% alcohol solution to give 20% gallons of 15% alcohol solution.

I need and explanation on how to set it up and do it. I have a test today and I don’t understand it.

Answers

Answer:

2.5 gallons of the 20% solution must be replaced by the 36% solution.

Step-by-step explanation:

[tex].12(20 - x) + .36x = 20(.15)[/tex]

[tex]2.4 - .12x + .36x = 3[/tex]

[tex]2.4 - .24x = 3[/tex]

[tex] - .24x = - .6[/tex]

[tex].24x = .6[/tex]

[tex]x = 2.5[/tex]

Given the following formula,A school needs to buy new notebook and desktop computers for its computer lab. The notebook computers cost $300 each, and the desktop computers cost $225 each. How much would it cost to buy 18 notebooks and 12 desktop computers? How much would it cost to buy n notebooks and dd desktop computers?

Total cost, 18 notebooks and 12 desktop computers:

Total cost, n notebooks and dd desktop computers:

Answers

The cost to buy 18 notebooks and 12 desktop computers will be $8100.

The cost to buy n notebooks and d desktop computers will be 300n + 225d

What is the cost of a given material?

The cost of a given material is the amount attached and to be paid for getting that material.

From the given information:

The cost of notebook computers = $300The cost of desktop computers is $225 each

The cost to buy 18 notebooks and 12 desktop computers will be:

= 300(18) + 225(12)

= 5400 + 2700

= $8100

The cost to buy n notebooks and d desktop computers will be:

= 300(n) + 225(d)

= 300n + 225d

Learn more about determining the cost of material here:

https://brainly.com/question/21720717

#SPJ1

What is m∠C?

Enter your answer in the box.

Answers

Answer:

60

Step-by-step explanation:

Total degree of triangle interior angles is 180

So x+2x+3x=180

6x=180

x=30

Measurement of angle C is 2x which is 60 degree

How tall is the tree?
5ft
35°
-20ft-
[? ]ft

Answers

Answer:

[tex]\boxed{\bf 19ft}[/tex]

Step-by-step explanation:

[tex]\bf tan\:\theta =\cfrac{opp}{adj}[/tex]

[tex]\bf tan(35^o)=\cfrac{x}{20}[/tex]

[tex]\bf 20\;tan (35^o)=x[/tex]

[tex]\bf 14ft=x[/tex]

The height of the tree:-

[tex]\bf x+5ft[/tex][tex]\bf 14ft+5ft[/tex][tex]\bf 19ft[/tex]

Answer:

19.00 ft to the nearest hundredth.

Step-by-step explanation:

tan 35 = h / 20

h = 20 tan 35

Now we add the man's height

Total height of the tree

=  20 tan 35 + 5

=  19.00 ft to the nearest hundredth.

Complete the tasks to subtract the polynomials vertically. (1.3t3 0.4t2 – 24t) – (0.6t2 8 – 18t) what is the additive inverse of the polynomial being subtracted?

Answers

The difference of the polynomials is [tex]1.3t^{3}-0.2t^{2}-6t-8[/tex] and the additive inverse of the polynomial being subtracted is [tex]-0.6t^{2} +8+18t[/tex].

The given polynomials are [tex](1.3t^{3}+0.4t^{2} -24t )[/tex] and [tex]0.6t^{2} +8-18t[/tex].

We need to find the additive inverse of the polynomial being subtracted.

What is additive inverse?

In mathematics, the additive inverse of a number a is the number that, when added to a yields zero. This number is also known as the opposite, sign change, and negation.

Now, [tex](1.3t^{3}+0.4t^{2} -24t )-(0.6t^{2} +8-18t)[/tex]

[tex]=1.3t^{3}+0.4t^{2}-0.6t^{2}-24t+18t-8[/tex]

[tex]=1.3t^{3}-0.2t^{2}-6t-8[/tex]

The additive inverse of the polynomial being subtracted is [tex]-0.6t^{2} +8+18t[/tex].

Therefore, the difference of the polynomials is [tex]1.3t^{3}-0.2t^{2}-6t-8[/tex] and the additive inverse of the polynomial being subtracted is [tex]-0.6t^{2} +8+18t[/tex].

To learn more about the additive inverse visit:

https://brainly.com/question/13715269.

#SPJ1

#+3)(2+1) for x = 3.
c+5)(x-5)

Answers

Answer:

x=1/3

Step-by-step explanation:

first multiply both bracket to together and then you get, 9x=3

x=1/3, then sub in to other equation and get the rest answer.

hope it helpful

Solve equation by using the quadratic formula. 6x^2 +2x =4?
A) x= 3/2, 1 B) x= 2/3, -1 C) x=3/2, -1 D) x=3/2, 0

Answers

Answer:

Answer x=2/3, -1
Option B

Step-by-step explanation:

Answer:

B) x =( 2/3, -1)

Step-by-step explanation:

Equation:

[tex]6x {}^{2} + 2x = 4[/tex]

Solution:

We know that,

[tex] \rm \: Quadratic \: formula (x)=\cfrac{-b\pm\sqrt{b^2-4ac}}{2a}[/tex]

According to the problem,

a = 6b = 2c = -4

Plugging them on the formulae,we obtain,

[tex]x = \cfrac{ - 2 \pm \sqrt{ 2 {}^{2} - 4 \{6 \times ( - 4) \} } }{2 \times 6} [/tex]

[tex]x = \cfrac{ - 2 \pm \: \sqrt{4 - 4 \{ - 24 \} } }{12} [/tex]

[tex]x = \cfrac{ - 2 \pm \: 10 }{12} [/tex]

[tex]x = \cfrac{ - 1 \pm5}{6} [/tex]

[tex]\boxed{x = \cfrac{2}{3} \: \rm or - 1}[/tex]

Choice B is accurate.

This is confusing please help

Answers

Step-by-step explanation:

since we are using Pythagoras

c² = a² + b²

with c being the Hypotenuse (the baseline opposite of the 90° angle),

you want to get the expression

y² = 3² + 3²

so, you pick the ² and put it next to the y left of the "=" sign.

and to the right of the "=" sign you put 3, ², +, 3, ²

The table represents the function f(x).

A 2-column table with 7 rows. The first column is labeled x with entries negative 4, negative 3, negative 2, negative 1, 0, 1, 2. The second column is labeled f of x with entries negative 66, negative 29, negative 10, negative 3, negative 1, 6.

When f(x) = –3, what is x?

Answers

Answer:

-1/2

Step-by-step explanation:

Answer:x is -1

Step-by-step explanation:

20.
(05.07 MC)

A scatter plot is made to model the amount of money left to pay on a mortgage. The data used for the scatter plot are shown in the table:

Number of Months 3 5 1 2 0 4
Money Owed $114,000 $100,000 $118,000 $116,000 $120,000 $112,000


What does the y-intercept of the model represent? (1 point)

The amount of the mortgage that has been paid that month

The monthly payments

The original amount of the mortgage

The number of months it takes to pay off the mortgage

Answers

Option third "The original amount of the mortgage" is correct because every month that passes, the amount left to pay decreases.

What is the line of best fit?

A mathematical notion called the line of the best fit connects points spread throughout a graph. It's a type of linear regression that uses scatter data to figure out the best way to define the dots' relationship.

[tex]\rm m = \dfrac{n\sum xy-\sum x \sum y}{n\sum x^2 - (\sum x)^2}[/tex]

[tex]\rm c = \dfrac{\sum y -m \sum x}{n}[/tex]

As we know the line of best is represented by:

y = mx + c

Here m is the slope and c is the y-intercept.

A scatter plot is made to model the amount of money left to pay on a mortgage.

Every month that passes, the amount left to pay decreases.

Plug x = 0 to get the y-intercept

y = c

The y-intercept representing the original amount of the mortgage.

Thus, option third "The original amount of the mortgage" is correct because every month that passes, the amount left to pay decreases.

Learn more about the line of best fit here:

brainly.com/question/14279419

#SPJ1

Which of the following values can be found by using the formula √1+tan^2 0?

Answers

√1+tan²Ø√1+sin²Ø/cos²Ø√cos²Ø+sin²Ø/cos²Ø√1/cos²Ø1/cosØsecØ

cos and sec can be found

The requried simplified expression is 1 / cosØ, which is equivalent to secØ. The correct option is secØ.

Let's simplify the given expression and see which of the provided options match:

We are given the expression √(1 + tan²Ø).

Now, recall the trigonometric identity: tan²Ø = sin²Ø / cos²Ø.

Substitute this identity into the expression:

√(1 + sin²Ø / cos²Ø)

Next, find a common denominator for the fraction inside the square root:

√((cos²Ø + sin²Ø) / cos²Ø)

Since cos²Ø + sin²Ø = 1 (as per the Pythagorean trigonometric identity), the expression becomes:

√(1 / cos²Ø)

Now, we can take the square root of 1 and cos²Ø separately:

√1 / √cos²Ø

Simplify:

1 / cosØ

The simplified expression is 1 / cosØ, which is equivalent to secØ.

Therefore, the correct option is: secØ.

Learn more about trigonometry here:

https://brainly.com/question/20885025

#SPJ4

The average of three unique number is 50.if the loweat number is 30 what is the sum of other two numbers

Answers

Answer:

The sum of the other two numbers is 120.

Step-by-step explanation:

To find an average, you add all the numbers involved and divide it by the amount of numbers involved. in this case,

(x+y+z) ÷ 3 =50

x=30

Divide both sides by three to remove the denominator.

(30+y+z)÷ 3 (*3) = 50(*3)

(30+y+z)=150

Subtract 30 from both sides.

(30+y+z)-30= 150-30

y+z=120

help me

A rectangle has a length of 10.3 centimeters and a width of 6.8 centimeters.

What is the area of this rectangle?


34.2 cm²

35.02 cm²

70.04 cm²

108.24 cm²

Answers

Answer:

70.04 cm²

Step-by-step explanation:

Area = base x height

10.3 x 6.8 = 70.04

Hope this helps :)

ATTACHED IS THE THE SOLUTION!!!

If the nth term of a sequence is 18-8n what is the 8th term

Answers

Answer:

[tex]a_{n} =18-8n\\a_{8} =18-8(8)=-46[/tex]

Answer:

-46

Step-by-step explanation:

(hello, I just answered your previous question--some stuff will sound similar because we are applying the same concept :)  )

We know that the nth term of the sequence is 18 - 8n

the "nth" term of a sequence means that you can plug in the term place to get the term value (ex. if the nth term is n + 2, we can plug any term in {let's say, the eighth term, 8 + 2 = 10 ; to get our term's value}  )

  18 - 8n is sequence

  18 - 8(8) is 8th term

  18 - 64 is 8th term

 -46 is the 8th term

hope this helps!! :)

Solve using substitution method:
X= y/3 +2
7x − 5y = 10

Solve for (x,y)

Answers

[tex]\quad \huge \quad \quad \boxed{ \tt \:Answer }[/tex]

[tex]\qquad \tt \rightarrow \:x = 5/2 [/tex]

[tex] \qquad \tt \rightarrow \: y = 3/2[/tex]

____________________________________

[tex] \large \tt Solution \: : [/tex]

[tex]\qquad \tt \rightarrow \: x = \cfrac{y}{3} + 2 \: \: \: \: \: \: \: \: - (1)[/tex]

[tex]\qquad \tt \rightarrow \: 7x -5 y = 10 \: \: - (2)[/tex]

[tex] \textsf{put value of x in equation 2} [/tex]

[tex]\qquad \tt \rightarrow \: 7 \bigg( \cfrac{y}{3} + 2 \bigg) - 5y = 10 [/tex]

[tex]\qquad \tt \rightarrow \: \cfrac{7y}{3} + 14 - 5y = 10 [/tex]

[tex]\qquad \tt \rightarrow \: \cfrac{7y}{3} - 5y = 10 - 14[/tex]

[tex]\qquad \tt \rightarrow \: \cfrac{7y - 15y}{3} = - 4[/tex]

[tex]\qquad \tt \rightarrow \: - 8y = - 4(3)[/tex]

[tex]\qquad \tt \rightarrow \: y = \cfrac{ - 12}{ - 8} [/tex]

[tex]\qquad \tt \rightarrow \: y = \cfrac{ 3}{ 2} [/tex]

[tex] \textsf{put value of y in equation 1 } [/tex]

[tex]\qquad \tt \rightarrow \: x = \cfrac{y}{3} + 2[/tex]

[tex]\qquad \tt \rightarrow \: x = \bigg (\cfrac{3}{2} \sdot\cfrac{1}{3} \bigg) + 2[/tex]

[tex]\qquad \tt \rightarrow \: x = \cfrac{1}{2} + 2[/tex]

[tex]\qquad \tt \rightarrow \: x = \cfrac{1 + 4}{2} [/tex]

[tex]\qquad \tt \rightarrow \: x = \cfrac{5}{2} [/tex]

Find the sum of the largest and the smallest of the following four numbers:
3.09 3.9 0.39 0.399


PLEASE HELP

Answers

Answer:

Using the concepts of Number System, we find that the sum of the largest and the smallest of the given four numbers is 4.29.

Step-by-step explanation:

The given set of numbers is 3.09, 3.9, 0.39 and 0.399.

The given numbers are related to each other as:

3.9 > 3.09 > 0.399 > 0.39

3.9 is greater than 3.09 as

3.9 > 3.09

3 + 0.9 > 3 + 0.09

0.9 > 0.09

90 > 9

0.399 is greater than 0.39 as

0.399 > 0.39

399 > 390

Now, the smallest number in the series is 0.39 and the largest number in the series is 3.9.

The sum of the largest and the smallest of the given four numbers is:

3.9 + 0.39

4.29

For more explanation, refer the following link:

https://brainly.com/question/9842110

#SPJ10

divided it by 7 and added
divided it by 7 and added 25. the result was 34 what was my number?
25. the result was 34 what was my number?

Answers

Answer:

your questions is not clear but from my observations

let the number be x

x/7+25=34

x+175/7=34

x+175=238

x=238-175

×=63

please mark me as brainlyst

The circle above with center O has a circumference of 36. What is the length of minor arc AC? A) 9 B) 12 C) 18 D) 36

Answers

Answer:

A) 9

Step-by-step explanation:

⇒ Circumference of a Circle = 2πr

[tex]36=2\pi r[/tex]

[tex]r=\frac{36}{2} \pi[/tex]

[tex]r=\frac{18}{\pi }[/tex]

Length of arc = rθ

θ = angle AC = [tex]90^o[/tex]

Convert [tex]90^o[/tex] to radian

= [tex]\frac{90\pi }{180}[/tex]

= [tex]\frac{\pi }{2}[/tex]

length of arc = 18/π x π/2

= 9

adam has 18 counters he gives 10 to lesleay how many of the 18 counters does lesleay get give your answer in simplest form

Answers

The fraction of the counters that Lesleay gets is 5/9.

What is the fraction?

A fraction is a quantity that is not a whole number. In maths, a fraction usually has a numerator and a denominator. An example is 1/2.

The fraction of the counters that Lesleay gets = number of counters she gets / total number of counters

10/18 = 5/9

To learn more about fractions, please check: https://brainly.com/question/915789

#SPJ1

pleasee help me with this function asap

Answers

Answer:

  b, c

Step-by-step explanation:

A function is continuous if its graph can be drawn without lifting the pencil. It is decreasing wherever its slope is negative.

__

A graph of the function is attached. It has a "jump" discontinuity at x=0, so is not a continuous function.

The value of f(0) is 2, so the y-intercept is 2.

The given function is defined for all values of x, so its domain is all real numbers.

The function is decreasing for values of x > 0, so does not approach positive infinity for large positive x.

The function has a stationary point at x=0, so is not decreasing over its entire domain.

_____

Additional comment

The function is decreasing everywhere except at x=0. The point (0, 2) is the vertex of the quadratic portion of the function, so a tangent is horizontal there. At such horizontal tangent points, a function is neither increasing nor decreasing. It is tempting to ignore this exception, because the function is decreasing everywhere else.

(06.06)

Which of the following demonstrates the Commutative Property of Multiplication?

5(2a − 3) = 10a − 15
10a − 15 = (2a − 3) ⋅ 5
5(2a − 3) = (2a − 3) ⋅ 5
(5 ⋅ 2a) − 3 = 5(2a − 3)

Answers

The correct answer is option C which is 5(2a − 3) = (2a − 3) ⋅ 5 showing commutative property.

What is commutative property?

If altering the operands' order has no effect on the outcome, the binary operation is commutative in mathematics.

a x b = b x a

So for the expression 5(2a − 3) = (2a − 3) ⋅ 5 suppose 5 is a and ( 2a - 3 ) is b so.

a x b = b x a

5(2a − 3) = (2a − 3) ⋅ 5

Therefore the correct answer is option C which is 5(2a − 3) = (2a − 3) ⋅ 5 showing commutative property.

To know more about commutative property follow

https://brainly.com/question/2475734

#SPJ1

The zeros of a parabola are -4 and 2 and (6,10) is a point on the graph which equation can be solved to determine the value of a in the equation of the parabola

Answers

Answer:

Let consider the following linear equation systems by using the known points and second-grade polynomial:

(-1, 7)

a + b + c = 7a+b+c=7

(5, 7)

25\cdot a + 5\cdot b + c = 725⋅a+5⋅b+c=7

(6,10)

36\cdot a + 6 \cdot b + c = 1036⋅a+6⋅b+c=10

After some algebraic manipulation, the values for the polynomial coefficients are found:

a = \frac{3}{5}a= 53 , b = -\frac{18}{5}b=− 518

, c = 10c=10

The polynomial is:

y = \frac{3}{5}\cdot x^{2} - \frac{18}{5}\cdot x+10y= 53

Step-by-step explanation:

For complete solution see the above attachment!

Does the chart below represent a proportional relationship? What is the rate of change?

x
y
1
4
2
7
3
10
4
13

Answers

Answer:

C will be your answer.

Please Mark as brainlist!

The line ax − by + 3 = 0 is parallel to the line 3x + 2y − 4 = 0 and passes through the point (1, −2). Find a and b.

Answers

Answer:

a=9

b=-6

Step-by-step explanation:

Give the values of a, b, and c needed to write the equation's general form.
(x - 3)2 = 0

1. a = 1; b = -3; c = 0
2. a = 1; b = -6; c = -3
3. a = 1; b = -6; c = 9

Answers

The values of a, b, and c are needed to write the equation's general form will be a = 1; b = -3; c = 0.Option A is corect.

What is the equation?

A mathematical statement consisting of an equal symbol between two algebraic expressions with the same value is known as an equation.

Given equation;

x²-3x=0

The quadratic equation in the form is;

ax² + bx + c = 0

On putting the values of the options one by one we will get the answer;

1 x²+(-3)x+0

x²-3x=0

Hence, option A is corect.

To learn more, about equations, refer;

https://brainly.com/question/10413253

#SPJ1

Other Questions
The TCP\IP protocol was invented by who Round to the nearest hundredth48.849 Which activity involves gross motor skills?Group of answer choicesthrowing a baseballtying a shoelacezipping a jacketcoloring alguien que me ayude haciendo una parafrasis del libro de gabriel garcia marquez cronica de una muerte anunciada Two of the factors that could contribute to low voter turnout are: (Select twoanswers)Select one or more:a. scandals of the past several decadesb. voters immigrating to other countriesc. growing alienation among votersd. typhoons during election day If one side of the DNA ladder contains the following bases, what would the complementary base pairs be for each? what would happen if the upbeat was down and the upbeat was up which type of cells are spherical and contain energy molecules? Jacob traveled 171 miles in 3.8 hours. He wants to know how many miles he traveled in one hour, so he set up this problem:171 divided by 3.8Jacob started by multiplying the divisor and the dividend by 10. This is the new problem:1710 divided by 38Use long division to find the quotient. 0.454.545450 Which statement about Jim's character throughout The Adventures of Huckleberry Finn is accurate?O Jim was shown less and less as time went on.O Depictions of Jim did not change as time went on.O As time went on, Jim was shown as a fuller character.O As time went on, Jim was shown as more of a stereotype. When selling alcohol most states consider your establishment responsible for all the following except Samantha is 9 years younger than her brother Maurice. In 10years, Maurice will be 1.5 times older than his sister. Find theirpresent ages. Function h is a transformation of the parent exponential function. Which statement is true about this function? h(x)=2^x-5 Help !! What is the difference between a comedy and a tragedy? 2. Which domain represents the greatest risk and uncertainty to an organization? Which of the binomials below is a factor of this trinomial? x2+12x+31A. x+4 B.x-2 C. x+2 D. x-4 a help desk structured into levels or tiers of support uses a ____ support model. How many moles of NO2 form when63.25 g N2O5 decompose?2NO54NO + O The word-skipping technique involves:A. skipping to the end of a passage with unfamiliar words.B. focusing on the main idea of a passage.C. replacing unfamiliar words with synonyms from a thesaurus.D. distracting yourself with pictures and drawings. )If 18th February, 2030 falls on Monday then what will be the day on 18th February, 2040Who ever answers this quickly with explanation I will make then brainliest